Verstrickt oder unverstrickt?

Ich war etwas verwirrt, als ich an zwei verschränkte Fermionen dachte.

Angenommen, wir haben einen Hilbert-Raum, in dem wir zwei fermionische Orbitale haben a und b . Dann der Hilbertraum H 's Dimension ist gerecht 4 , da es von überspannt wird

{ | 0 , c a | 0 , c b | 0 , c a c b | 0 } ,
wo c ich sind die fermionischen Operatoren, die ein Fermion im Orbital erzeugen ich .

Angenommen, wir haben einen Staat c a c b | 0 . Wenn ich dann meinen Hilbert-Raum in zwei Teile aufteile, indem ich das Tensorprodukt der Hilbert-Räume jedes Orbitals betrachte, dh H = H a H b , dann kann mein Zustand geschrieben werden als c a | 0 a c b | 0 b , woraus offensichtlich ist, dass dieser Zustand entwirrt ist ( | 0 = | 0 a | 0 b ).

Jetzt dachte ich darüber nach, den Zustand zuerst quantisiert zu schreiben, dh eine Wellenfunktion. Lassen ϕ a ( r ) , ϕ b ( r ) seien die Wellenfunktionen der Orbitale a und b . Dann

ψ ( x 1 , x 2 ) = x 1 x 2 | c a c b | 0 = ϕ a ( x 1 ) ϕ b ( x 2 ) ϕ a ( x 1 ) ϕ b ( x 2 ) .
Hier wurde ich verwirrt. Welches Objekt ist ψ ( x 1 , x 2 ) , dh zu welchem ​​Hilbertraum gehört er? Was genau tun wir, wenn wir es tun x 1 x 2 | c a c b | 0 ? Wir scheinen unseren Hilbert-Raum zu ändern/erweitern, indem wir die Positionsdarstellung einnehmen?

Auf diese Weise geschrieben und unter der Annahme der gleichen Partition H a H b , ist die unverstrickte Natur des ursprünglichen Zustands nicht mehr sichtbar. Ich bin mir nicht sicher, was die Partition H a H b bedeutet in diesem Zusammenhang sogar. Soll das heißen ψ ( x 1 , x 2 ) = ψ a ( x 1 , x 2 ) × ψ b ( x 1 , x 2 ) wo ψ ich ( x 1 , x 2 ) ist eine Linearkombination von ϕ ich ( x 1 ) , ϕ ich ( x 2 ) ? Das erscheint mir nicht richtig.

Unabhängig davon habe ich jetzt einen Zustand, der auf zwei verschiedene, aber angeblich äquivalente Arten geschrieben ist, mit derselben Aufteilung des Hilbert-Raums, der jedoch auf die eine Weise entwirrt und auf der anderen verwickelt ist.

Hilfe?

Sie können einen Zwei-Fermionen-Sektor des Fock-Raums nicht als Tensorprodukt partitionieren - zugrunde liegende Zustände sind immer antisymmetrisch
@ Slaviks: Entschuldigung, ich habe nicht ganz verstanden, was Sie meinten. Sagst du H = H a H b ist falsch? aber so baut man Multi-Fermion-Fock-Zustände auf. jeder Betreiber c ich wirkt in einem Hilbert-Raum, der nur das Vakuum und eine Anregung ist, und so weiter N Fermionen H = ich = 1 N H ich ( 2 N mögliche Zustände). Hier überlege ich gerade N = 2 .
@Slaviks - Wenn Sie eine Jordan-Wigner-Transformation anwenden, erhalten Sie sofort eine Tensorproduktzerlegung in zweidimensionale Hilbert-Räume. Ich denke, diese Frage ist eine laufende Debatte in der Literatur. Da der Umgang mit der (Anti-)Symmetrisierung nicht ganz einfach zu sein scheint, gibt es unterschiedliche Ansätze. Die Antwort muss meines Erachtens durch die Betrachtung von Messergebnissen und deren Korrelationen und damit (wahrscheinlich) Faktorisierungseigenschaften von Ein- und Zweikörper-Dichtematrizen im einfachsten Fall gefunden werden. Siehe z. B. arXiv:0902.1684 und PRA 67, 024301 (2003) und ihre Referenzen.
@nervxxx Ich verstehe, was du meinst. Ich sage diese Zerlegung der Besetzungszahl H 1 H 2 ist nicht dasselbe wie die (unmögliche) Zerlegung in das Tensorprodukt von Ein-Teilchen-Hilbert-Räumen. Aber man muss noch mehr nachdenken, besonders angesichts des Kommentars von S. Gammelmark.
In der Tat gibt es viel Literatur darüber, dass die Antisymmetrisierung, die Sie automatisch in Multi-Fermion-Zuständen erhalten, nicht als operativ nützliche "Verschränkung" betrachtet werden sollte, z. B. Zanardi et al man kann seine konstituierenden physikalischen Freiheitsgrade manipulieren und untersuchen. In diesem Sinne ist Verschränkung immer relativ zu einer bestimmten Menge experimenteller Fähigkeiten.
@ Slaviks: immer noch verwirrt. Mein H ich ist der Hilbert-Raum einzelner Orbitale, kein Ein-Teilchen-Hilbert-Raum. Die Teilchenzahl ist unbestimmt (aber darauf beschränkt). 0 2 in diesem Fall), bis ich einen Zustand gebe. dh ich betrachte nicht den 2-Teilchen-Fock-Raum.
Um den Kommentar von twistor59 zu erweitern: In diesem Fall besteht eine wichtige Einschränkung darin, dass die Fermionen nicht unterscheidbar sind. In einem traditionellen Bell-Zustand können wir jede Messung an jedem System einzeln durchführen. Aber in einem ersten quantisierten Bild von 2 Fermionen, die 2 Modi belegen, gibt es keine Möglichkeit, eine Messung an nur einem der Fermionen durchzuführen. Es ist ein Vorteil des 2. quantisierten Bildes, dass diese scheinbare Einschränkung als Verwirrung dargestellt wird: Sie können nicht zwischen den beiden Fermionen unterscheiden, weil es sich nicht um zwei verschiedene Objekte handelt. Sie zählen nur die Erregung der grundlegenderen Objekte, der Moden.
@twistor59 +1 für die Zanardi-Referenz, spricht das Problem in Spalte 2 von Seite 1 gut an.

Antworten (3)

Ich möchte Sie daran erinnern, dass der Fock-Raum mehrerer Fermionen als antisymmetrischer (fermionischer) Unterraum des vollständigen Fock-Raums definiert ist

Γ a = n = 0 H n ,

wo steht für das antisymmetrische Tensorprodukt

v 1 v n = 1 n ! p P n σ p v p ( 1 ) v p ( n ) .

Hier σ p ist das Zeichen der Permutation p in der Gruppe der Permutationen P n .

Daher kommt die Verwirrung hier daher, dass c a c b | 0 | a b wie du zu sagen scheinst.

Erinnern Sie sich, dass die Erzeugungs- und Vernichtungsoperatoren innerhalb der Besetzungszahldarstellung definiert sind, d. h c a c b | 0 = | 11 , wobei die erste Zahl die Anzahl der Fermionen im Zustand angibt a während die zweite die Anzahl der Fermionen im Zustand angibt b . Andererseits sind Zustände, die in der Besetzungszahldarstellung geschrieben sind , als richtig antisymmetrisierte (für Fermionen) Vielteilchen-Basiszustände definiert, wie es uns durch die Ununterscheidbarkeit der Teilchen aufgezwungen wird. Daher sind sie im fermionischen Fockraum definiert. Jedes Lehrbuch wird dies zeigen, werfen Sie einen Blick auf das erste Kapitel von Many-Body Quantum Theory in Condensed Matter Physics: An Introduction von Bruus und Flensberg zum Beispiel. Für zwei Fermionen, die über eine einzelne Teilchenbasis beschrieben werden { | a , | b } eine mögliche Wahl ist:

| 11 = 1 2 ( | a b | b a ) .
Deswegen

c a c b | 0 = 1 2 ( c a | 0 1 c b | 0 2 c b | 0 1 c a | 0 2 )

Daraus ergibt sich nun die bekannte Antikommutativität dieser Operatoren

c b c a | 0 = 1 2 ( c b | 0 1 c a | 0 2 c a | 0 1 c b | 0 2 ) = c a c b | 0

Tatsächlich ist einer der großen Vorteile von Erzeugungs- und Vernichtungsoperatoren, dass sie die Antisymmetrie (für Fermionen) der Wellenfunktion implizit enthalten.

Punktieren Sie dies mit x 1 x 2 | wir erhalten:

x 1 x 2 | c a c b | 0 = 1 2 ( ϕ a ( x 1 ) ϕ b ( x 2 ) ϕ b ( x 1 ) ϕ a ( x 2 ) ) .

Es gibt also keinen Widerspruch, beide Darstellungen zeigen, dass die Teilchen verschränkt sind.

Auf der anderen Seite Punktierung x 1 x 2 | mit c a | 0 1 c b | 0 2 würde einfach produzieren

ψ ( x 1 , x 2 ) = ϕ a ( x 1 ) ϕ b ( x 2 )

Daher gibt es auch hier keine Widersprüchlichkeit, aber wie gesagt, das Wichtigste ist, sich daran zu erinnern

c a c b | 0 c a | 0 1 c b | 0 2

Nein das ist nicht richtig. Mit der von mir in der Aufgabe angegebenen Wahl der Aufteilung des Hilbert-Raums sind die beiden Teilsysteme eindeutig entflochten . Ich zitiere zum Beispiel die Verwendung des Orbitalschnitts im Verschränkungsspektrum von QH-Zuständen (sehen Sie es nach) - darin sind IQH-Zustände unverschränkt, weil sie die Form haben | ψ = ich c ich | 0 .
Nächste, c a c b | 0 = 1 2 ( c a | 0 1 c b | 0 2 c b | 0 1 c a | 0 2 ) ist falsch. Das Mehrteilchen-Fock-Zustandsvakuum wird aus dem Tensorprodukt einzelner Vakua aufgebaut ich | 0 ich . Dann entsteht eine Fock-Zustandsdarstellung, indem die Erzeugungsoperatoren darauf einwirken: c a c b | 0 = | 0 ich c a | 0 a c b | 0 b | 0 j
Die Antikommutativität entsteht aufgrund der kanonischen Antikommutierungsrelationen { c ich , c j } = δ ich j . Daraus ergibt sich dann c a c b | 0 = c b c a | 0 . Der springende Punkt bei der Darstellung der Fock-Zustandszahl ist, dass sie die Teilchen sofort als nicht unterscheidbar behandelt und keine unphysikalische Bezeichnung hat (die Positionen, x 1 , x 2 , ) für jedes Teilchen wie eine erste quantisierte Notation (Wellenfunktion). Die erste quantisierte Notation ist wirklich umständlich und umständlich, weil sie was bewirkt
bedeutet, dass jedes Partikel mit Positionskennzeichnungen unterscheidbar ist x 1 , x 2 , , aber dann erzwingt es die Regel, dass sie tatsächlich nicht unterscheidbar sind und unter einem Etikettentausch ungerade sein sollten, was zum Slater-Determinanten- / Antisymmetrisierungsschema führt. Also -1.
@nervxxx, ich habe meine Antwort bearbeitet, um meinen Standpunkt weiter zu erläutern. Ich möchte aber auch, dass Sie mir konkrete Hinweise geben, denn „nachschlagen“ ist keiner.
@nervxxx, was den Mehrteilchen-Fock-Raum betrifft, habe ich in meiner Antwort erklärt, dass man für Fermionen das antisymmetrische Tensorprodukt verwendet. Also für zwei Fermionen c a c b | 0 = 1 2 ( c a | 0 1 c b | 0 2 c b | 0 1 c a | 0 2 ) ist richtig.
Ich stimme dem Rest Ihrer Kommentare zu und weise darauf hin, dass meine gesamte Antwort darauf basiert.
Sie benötigen möglicherweise etwas Hintergrundwissen in QH-Physik, aber die v = 1 Der IQH-Zustand ist ein reiner Produktzustand princeton.edu/~jmaciejk/entanglement2012/slides/… Folie 8 und arxiv.org/pdf/1111.2810.pdf Seite 2 „Eine einfache Inspektion zeigt, dass der Grundzustand ρ ist ein Produktzustand in Orbitalbasis und daher ρ EIN ist ein reiner Zustand für jede orbitale Zweiteilung (wobei alle N EIN Orbitale im Subsystem EIN gefüllt sind) dieses Systems. Folglich ist die ES (und damit die Verschränkungsentropie) trivial und besteht aus genau einer Ebene.
Das Wesentliche ist, dass es für einen reinen Produktzustand bei der Art der Partitionierung, die ich in meinem Beitrag beschrieben habe, keine Verschränkung gibt, dh es hat nur einen Schmidt-Wert.
Lieber @nervxxx, vielen Dank für die Ressourcen. Ich verstehe und stimme diesen Argumenten zu. Der einzige Punkt, den ich machen wollte, ist, dass, wenn Sie a wollen H = H a H b Partition, die Sie nicht verwenden können c a c b | 0 Vektor, da er eine antisymmetrische Partition impliziert. Das ist, glaube ich, die Quelle der Verwirrung.
diese aussage entbehrt jeder grundlage. Warum kannst du den Vektor nicht verwenden? Sie wählen einen Vektor einfach aus dem vollen Hilbert-Raum H . Die Aufteilung, die Sie dem Hilbert-Raum auferlegen, ist eine völlig unabhängige Wahl. Für die Ressourcen, die ich Ihnen gegeben habe, schauen Sie: Auf der niedrigsten Landau-Ebene ist der Hilbert-Raum durch das Tensorprodukt der Orbital-Hilbert-Räume gegeben H = ich = 1 l H ich . Der Orbitalschnitt ist durch die Wahl der Teilung gegeben H = H a H b ( ich = 1 m H ich ) ( ich = m + 1 l H ich ) . Der IQH-Zustand, der gegeben ist durch
( ich = 1 l c ich ) | 0 , unter dieser Partitionierung, wird ich = 1 m c ich | 0 ich = m + 1 l c ich | 0 . Es gibt also keine Verstrickung. Das meinen die Autoren damit, dass der Orbitalschnitt nur ein Schmidt-Niveau hat. Ich glaube stattdessen, dass Sie mit Ihrem Verständnis des antisymmetrisierten Fock-Raums ziemlich verwirrt sind.
Ich würde es begrüßen, wenn Sie angeben könnten, was Ihrer Meinung nach an meiner Antwort falsch ist. Wollten Sie das sagen c a c b | 0 impliziert nicht das antisymmetrische Tensorprodukt?
Beachten Sie, dass die Art und Weise, wie Sie die Frage gestellt haben, sehr wenig mit IQH zu tun hat. Es geht auch nicht auf die Natur der Verschränkung ununterscheidbarer Teilchen ein, die zB in physik.rwth-aachen.de/fileadmin/user_upload/www_physik/… und sqig.math.ist.utl.pt/pub/PaunkovicN/04 diskutiert wird -P-Doktorarbeit.pdf
Außerdem scheint mir, dass die Frage, die Sie gestellt haben, nicht die ist, die Sie stellen wollten, und die Antwort auf letztere könnte hier sein : physical.stackexchange.com/questions/35185/…
Der Bachelorarbeit, die Sie mir gegeben haben, würde ich nicht trauen - sie hat Fehler. die Doktorarbeit ist ok. Meine Frage hat alles mit dem IQH zu tun - ich beschränke die Anzahl der Orbitale nur zur Veranschaulichung auf 2, während im IQHE die Anzahl der Orbitale auf einem kompakten Verteiler liegt N Ö r b . Ok, was an Ihrem Kommentar falsch ist, wird in meinem zweiten Kommentar in der Kette angegeben. Ihre letzte Behauptung, das c a c b | 0 das ist nicht richtig. Sie verwechseln die zweite quantisierte Notation vollständig mit der ersten!
Aber trotzdem sehe ich das Problem jetzt, sorry, dass es so lange gedauert hat. Mein H ist nicht dasselbe wie dein H . Dein H ist der Ein-Teilchen-Hilbert-Raum. Mein H ich ist der Hilbertraum der ich -ter fermionischer Modus. Ich werde diesen Unterschied illustrieren, indem ich Argumente zähle. Sagen Sie, dass wir haben M Orbitale (mit unterschiedlichen Quantenzahlen). Jedes Orbital hat seine zugeordneten Hebe- und Senkoperatoren c ich , c ich . Der Hilbert-Raum des Orbitals (den ich verwendet habe) hat also die Dimension 2. Der vollständige Hilbert-Raum ist das Tensorprodukt von M Orbitale, ich H ich , die schwach ist 2 M .
Dieses Tensorprodukt ist weder symmetrisiert noch antisymmetrisiert. Es ist ein einfaches Tensorprodukt. Schauen wir uns jetzt IHRE an H , die wir nennen H 1 , der Einteilchen-Hilbert-Raum. Es wird überspannt M mögliche Zustände ϕ 1 ( x ) , ϕ M ( x ) . Wir können höchstens haben M Elektronen in diesem System, also muss die Dimension des vollen Hilbert-Raums endlich sein. Zählen wir es explizit. Welche Dimension hat der 2-Teilchen-Hilbert-Raum? Ist es H 1 H 1 ? Nein, weil wir die erste quantisierte Notation verwenden und um dem Spin-Statistik-Theorem zu gehorchen, müssen wir diesen Raum antisymmetrisieren.
Das gibt uns H 2 = S v H 1 H 1 H 1 H 1 , wo S v ist der Antisymmetrierungsoperator auf einem Tensor. Was ist die Dimension von H 2 ? Es ist ( M 2 ) . Dann ähnlich dimmen H 3 = ( M 3 ) , und so weiter, bis H M . Schließlich stellen wir fest, dass es gibt H 0 , das Vakuum, dem KEINE Wellenfunktion zugeordnet ist (keine Argumente!) Und die Dimension 1 hat. Dies definiert den Fock-Raum, wie Sie gesagt haben, H = H 0 H 1 H M , die schwach ist ( M 0 ) + ( M 1 ) + + ( M M ) = 2 M , das gleiche wie vorher!
Der Hauptunterschied liegt jedoch darin, was H eigentlich bedeutet. Lassen Sie sich nicht verwirren. Denken Sie daran, meine H ist der Orbital-Hilbert-Raum, der schwach ist 2 . Ihr Einzelteilchen-Hilbert-Raum ist schwach M , die möglicherweise unendlich sein kann. Die Erzeugungs- und Vernichtungsoperatoren ergeben die sogenannte zweite Quantisierungsformulierung von QM, die das Problem der Antisymmetrierung in ihren Kommutierungsbeziehungen vollständig vermeidet (oder vielmehr verbirgt). Das ist, wo Sie falsch liegen, weil Sie verwenden c a , c b wie in erster quantisierter Notation.
Ich habe nicht die Geduld, den gesamten Austausch zu lesen, aber ich frage mich, ob die Verwirrung davon abhängt, welche Art von Partitionierung Sie vornehmen . In diesem Fall werde ich versuchen, zu kommentieren. Das v = 1 Der QH-Zustand ist rein unter orbitaler Partitionierung, aber nicht unter "Partikelpartitionierung" . Vielleicht hilft das -- arxiv.org/abs/0905.4024 @nervxxx, Ihr 2-Teilchen-Zustand könnte unter orbitaler Partitionierung rein sein, aber er ist unter Partikelpartitionierung verwickelt. Die Verschränkung hängt vollständig davon ab, wie Sie Ihr System partitionieren.
Ich habe diesen Kommentar in meiner Antwort ausgearbeitet.

Ich poste eine modifizierte Version meines Kommentars als Antwort, da mehr Leute es so sehen werden.

Ich denke, die Verwirrung hängt entscheidend davon ab, welche Art von Partitionierung Sie durchführen . Das v = 1 Der QH-Zustand ist unter Orbitalpartitionierung rein, aber nicht unter "Partikelpartitionierung". Vielleicht hilft arXiv:0905.4204 . IIRC, sie erarbeiten im 2. Abschnitt ein einfaches Beispiel zu diesem Detail.

@nervxxx, Ihr 2-Teilchen-Zustand mag unter orbitaler Partitionierung rein sein, aber er ist unter Partikelpartitionierung verwickelt. Aufgrund der Antisymmetrisierung sieht es aus wie ein Singulett-Bell-Zustand.

Das Fazit ist also, dass die Verschränkung vollständig davon abhängt, wie Sie Ihr System partitionieren. Die Subtilität wird nicht allgemein geschätzt. Eine differenzierte Diskussion finden Sie in diesem Artikel http://rspa.royalsocietypublishing.org/content/463/2085/2277.full

Ja dank. Das habe ich schließlich festgestellt. Aber was mich ärgert (und der Grund für den ganzen langen Austausch oben) ist, dass fast alle anderen den Fock-Raum nicht verstehen (mit Einzelpartikel-Hilbert-Raum von dim M ) ist isomorph zum Tensorprodukt von M orbitale Räume. Man kann dies sehen, wenn man sich die Dimension der Hilbert-Räume ansieht – beide sind es 2 M , und man hat eine 1-1 zwischen den beiden Leerzeichen, also sind sie isomorph! Das impliziert, dass man den Fock-Raum tatsächlich in ein Tensorprodukt zerlegen kann - außer dass, weil dieser Isomorphismus nur für die vollen Räume funktioniert,
man kann die Singulett-Wellenfunktion nicht in ein Tensorprodukt zerlegen wie c a | 0 c b | 0 und stellen Sie es als einige dar f ( x 1 ) g ( x 2 ) .

Wir müssen mit dem Braket-Formalismus und seiner Bedeutung vorsichtig sein. nicht wie | x 1 , Ich bin mir nicht sicher, ob die Notation | x 1 x 2 wo x 1 und x 2 sind Positionskoordinaten sinnvoll. In der Literatur [1] die Notation | a b bezeichnet die Slater-Determinante oder den Hartree-Fock-Zustand, dh:

| a b = c a c b | 0 = ϕ a ( x 1 ) ϕ b ( x 2 ) ϕ a ( x 2 ) ϕ b ( x 1 )

Mein Gefühl ist, dass Ihre Verwirrung mit der Vermischung des Besetzungszahlenformalismus und der Realraumdarstellung zusammenhängt.

[1] Szabo, Ostlund, "Moderne Quantenchemie: Einführung in die fortgeschrittene Theorie der elektronischen Struktur"

Wenn jemand erklärt, was hinter der Notation |r1 r2> steht, wäre ich sehr dankbar.
| x 1 x 2 ist eine gebräuchliche Abkürzung für | x 1 | x 2 . Im Allgemeinen ist das, was in einem Ket geschrieben steht, nur ein Etikett: Seine Bedeutung ist stark kontextabhängig. Das Mischen der beiden Darstellungen ist nicht ungewöhnlich, solange es keine Verwechslung gibt.
ok dem stimme ich voll und ganz zu. Aber betrachten wir einen Spezialfall aus der obigen Frage, nämlich den Fall, dass x 1 und x 2 Quantenzahlen sind, die die Eigenzustände des Positionsoperators ˆ x bezeichnen . Was bedeutet die Zerlegung von \ket x 1\ket x 2 ? Außerdem bin ich neugierig, ob es Referenzen gibt, die eine solche Notation für den Koordinatenraum enthalten. Es wäre interessant, mehr darüber zu lesen.
Ich habe x 1 und x 2 als Bezeichnung für die Position der beiden Fermionen verstanden. Aber natürlich ist das Ket \ket x 1 x 2 nicht antisymmetrisiert und entspricht keinem erlaubten Zustand. Nur \ket x 1 x 2\ket x 2 x 1 ist ein gültiger Zustand.
Bedeutet Kennzeichnung Quantenzahlen? Ich denke, Symbole in Bra- und Ket-Vektoren in der Dirac-Notation können nicht nur Etiketten sein. Sie sollten echte Quantenzahlen sein. Deshalb sehe ich keinen klaren Sinn in | x 1 x 2 > es sei denn, jemand erklärt mir, was der Operator | sagt x 1 x 2 > gehören?
Im Allgemeinen sind sie Beschriftungen für echte Vektoren im Hilbert-Raum. Wenn „echte Quantenzahlen“ „Etiketten zur Eigenbasis eines gegebenen Operators“ bedeuten, dann müssen sie nicht „echte Quantenzahlen“ sein. Wenn die Basis frei ist, dann weiß ich nicht genau, was "richtig" bedeutet
Bei Fermionen wird die Situation dadurch kompliziert, dass Sie eine globale Symmetrie auferlegen. | x 1 x 2 | ↑↑ ist kein gültiger Zustand, aber | x 1 x 2 ( | ↑↓ | ↓↑ ist